Assorted changes to get problems working with new, flexible labels.
[course.git] / latex / problems / Young_and_Freedman_12 / problem23.60.tex
1 \begin{problem*}{60}
2 A small sphere with mass $1.50\U{g}$ hangs by a thread between two
3 parallel vertical plates $5.00\U{cm}$ apart (Fig.~23.36).  The plates
4 are insulateing and have uniform surface charge densitied $+\sigma$
5 and $-\sigma$.  The charge on the sphere is $q=8.90\E{-6}\U{C}$.  What
6 potential difference between the plates will cause the thread to
7 assume an angle of $30.0\dg$ with the vertical?
8 \begin{center}
9 \begin{asy}
10 import Mechanics;
11 import ElectroMag;
12
13 real u = 0.3cm;
14 real L = 4;
15 real ds = 5;
16 real phi = 30;
17 real dy = 1.8*L*Cos(phi);
18 int n=3;
19
20 Charge q = pCharge(dir(-90+phi)*L*u, q=1, L="$q$");
21 Wire wire = Wire((0,0), q.center());
22 Angle theta = Angle(q.center(), (0,0), (0,-1), L="$\theta$");
23 Surface s = Surface((ds/2,0)*u, (-ds/2,0)*u);
24 Wire right_plate = Wire((ds,0)*u, (ds,-dy)*u, outline=green);
25 Wire left_plate = Wire((-ds,0)*u, (-ds,-dy)*u, outline=green);
26 Distance Ds = Distance(left_plate.pTo, right_plate.pTo,
27                        L=Label("$5.00\U{cm}$", align=S));
28
29 left_plate.draw();
30 right_plate.draw();
31 s.draw();
32 Ds.draw();
33 wire.draw();
34 draw((0,0)--(0,q.center().y), dashed);
35 q.draw();
36 theta.draw();
37 \end{asy}
38 \end{center}
39 \end{problem*}
40
41 \begin{solution}
42 The sphere is stationary, so we know the forces must balance.
43 \begin{align}
44   \sum F_y &= T\cos\theta - mg = 0 \\
45   T &= \frac{mg}{\cos\theta} \\
46   \sum F_x &= qE - T\sin\theta = 0 \\
47   qE &= T\sin\theta = mg\tan\theta \\
48   E &= \frac{mg}{q}\tan\theta \;.
49 \end{align}
50
51 The $E$ field is constant, so the potential difference between the two
52 plates is given by
53 \begin{equation}
54   \Delta V = \int_\text{left}^\text{right} \vect{E}\cdot\vect{dx}
55     = E\int_\text{left}^\text{right} dx = Ed \;,
56 \end{equation}
57 and the potential difference between the plates is
58 \begin{equation}
59   \Delta V = Ed = \frac{mgd}{q}\tan\theta
60     = \frac{1.50\E{-3}\U{kg}\cdot9.80\U{m/s$^2$}\cdot5.00\E{-2}\U{m}}
61            {8.90\E{-6}\U{C}} \tan(30.0\dg)
62     = \ans{47.7\U{V}} \;.
63 \end{equation}
64 \end{solution}